An awesome number theory contest problem

Поділитися
Вставка
  • Опубліковано 20 вер 2024
  • 🌟Support the channel🌟
    Patreon: / michaelpennmath
    Merch: teespring.com/...
    My amazon shop: www.amazon.com...
    🟢 Discord: / discord
    🌟my other channels🌟
    Course videos: / @mathmajor
    non-math podcast: / @thepennpavpodcast7878
    🌟My Links🌟
    Personal Website: www.michael-pen...
    Instagram: / melp2718
    Randolph College Math: www.randolphcol...
    Research Gate profile: www.researchga...
    Google Scholar profile: scholar.google...
    🌟How I make Thumbnails🌟
    Canva: partner.canva....
    Color Pallet: coolors.co/?re...
    🌟Suggest a problem🌟
    forms.gle/ea7P...

КОМЕНТАРІ • 87

  • @talberger4305
    @talberger4305 2 роки тому +52

    It should be m+1 at 4:58

    • @krisbrandenberger544
      @krisbrandenberger544 2 роки тому

      That's exactly right ✅! And it would have been okay to prove by induction that m+1=2.

  • @jursamaj
    @jursamaj 2 роки тому +54

    Just for fun: the limit as n approaches -∞ is 1, which is a square.

    • @HershO.
      @HershO. 2 роки тому +2

      I suppose he missed a solution then lmao

    • @rocky171986
      @rocky171986 2 роки тому +1

      2^n yes, but not (n-1)*2^n, the limit is still -inf

    • @jursamaj
      @jursamaj 2 роки тому +27

      @@rocky171986 2^n grows much faster than n, so the division goes to 0, leaving the +1. That's a standard limit technique.

    • @DavidSavinainen
      @DavidSavinainen 2 роки тому +4

      @@jursamaj Alternatively,
      let m = -n, so the limit is -(m+1)2^(-m) as m approaches infinity.
      = -(m+1)/(2^m) as m approaches infinity
      L’Hopital now gives -1/(ln(2)2^m), which clearly approaches 0 as m approaches infinity.

    • @jursamaj
      @jursamaj 2 роки тому

      @@DavidSavinainen Yup, different method, same result.

  • @carstenmeyer7786
    @carstenmeyer7786 2 роки тому +16

    9:00 The factors *m* and *m+1* are relatively prime, so we can combine both cases:
    - *2^{n-2}* divides either *m* or *m+1* . In both cases, we have *2^{n-2} ≤ m+1*
    - The other factor divides *n-1* . In both cases, we have *m ≤ n - 1*
    For both statements to be true, we need to have *2^{n-2} ≤ n* . Via induction this is only true for *n ≤ 4*

  • @davidcroft95
    @davidcroft95 2 роки тому +77

    "n equals 1 is zero" Michael Penn, 2022

  • @think_logically_
    @think_logically_ 2 роки тому +4

    From 1-n

  • @udic01
    @udic01 2 роки тому +7

    4:48 1-n=1+m not m-1. Although the proof is the same

    • @TedHopp
      @TedHopp 2 роки тому

      Actually, it should have started with 0 < -1-n < 2^n. So the error was earlier and m-1 is correct.

  • @goodplacetostop2973
    @goodplacetostop2973 2 роки тому +13

    5:17 Classic Michael Penn induction
    14:03 Good Place To Stop

  • @Roberto-REME
    @Roberto-REME 2 роки тому +2

    You're the best, Michael!!!

  • @Gninofisico
    @Gninofisico 2 роки тому +4

    I have a little question: (n-1)2^n + 1 has to be an integer? Because it could be, for example, a number like 16/49 that is a perfect square.

  • @erazorheader
    @erazorheader 2 роки тому +3

    A bit easier way is to look at (n - 1)*2^n as a product of two consecutive even numbers:
    (n - 1)*2^n = a*(a+2), n >=2.
    Then, there are two cases depending on which one of a, a + 2 is divisible by 4.
    Case 1: a = b * 2^r, b is odd, r > 1. Here r > 1 because a is divisible by at least 4. Then,
    a + 2 = 2 * (b * 2^(r-1) + 1)
    Then, we see that
    (n - 1) * 2^n = 2^(r + 1) * b * (b * 2^(r - 1) + 1)
    From this it is clear that
    n = n - 1
    We also use that b >= 1, so we have the inequality:
    (n - 1) * 2^n >= 2^n * (2^(n-2) + 1)
    This can be simplified to
    n - 2 >= 2^(n-2)
    This inequality is false at any n >=2.
    Case 2: a + 2 = b * 2^r, b is odd, b >= 1, r >=2.
    After using similar logic we get:
    n >= 2^(n - 2)
    This is only possible for n = 2, 3 or 4.
    Then, by substitution we find that only n = 4 yields the solution among n >=2.

    • @backyard282
      @backyard282 2 роки тому

      yes but how do you know that (n-1)*2^n is a product of two consecutive even numbers?

    • @Idran
      @Idran 2 роки тому

      @@backyard282 assume (n-1)*2^n + 1= m^2
      then (n-1)*2^n = m^2 - 1 = (m-1)(m+1)
      since we know that m^2 has to be odd (as the video showed), then m must also be odd, and so m-1 and m+1 are the two consecutive even numbers on either side of m

    • @fordiscordfordiscord69
      @fordiscordfordiscord69 8 місяців тому

      Cool

  • @txikitofandango
    @txikitofandango 2 роки тому +3

    I've basically figured it out... I have
    (n - 1)2^(n-2) = k(k+1) for some integer k. Now either k or k+1 must be divisible by 2^(n-2), because 2 does not divide the other factor. This works out for n=4, but if n starts getting too big...
    For big enough n, n-1 < 2^(n-3), so the LHS < 2^(2n-5). However, either k or k+1 will be divisible by 2^(n-2), and therefore k > 2^(n-2), and the RHS > 2^(2n-4) so we've reached a contradiction. The RHS quickly outpaces the LHS. This is very crude but it works even if n is still quite small.
    Like, if n=8, then the LHS is 7*2^6, which means k >= 2^6, which means k(k+1) >= 2^12, and this discrepancy grows as n grows.

    • @leif1075
      @leif1075 2 роки тому

      What about breaking it into difference of squares k minus 1 times k pkus q and set each factor equal to 2^n or (n-1)..you can solve that way

  • @givrally7634
    @givrally7634 2 роки тому +5

    About the negative case, I wonder what happens if instead of integer perfect squares, we're talking about rational perfect squares ? By that, I mean rational numbers whose square roots are also rational. 4/9 is a rational perfect square but 1/2 isn't.

  • @nodiceism
    @nodiceism Рік тому +2

    Thanks for the videos. I have watched keenly and enjoyed. Getting good enough to actually finish some of them and I felt super accomplished having gotten to the end of this one! I prefer my solution though for case n>=5 although do appreciate that yours is more algebraically robust.
    If we accept that 2^(n-2)>n-1 (and the difference differs by more than 1) for n>=5. We can exhaustively extract all 2 contained in n-1 leaving an odd number. The LHS is now the product of two numbers: one is an odd number less than n-1 and the other a power of two that is greater than or equal to n-2. i.e LHS is now the product of two coprime integers. This implies that the smaller of these numbers is equal to smaller of that on the RHS and vice versa. However, the difference between these two numbers is even greater than what we started with (at least 2) and yet the RHS only differs by 1 which is a contradiction!

  • @sergeipetrov5572
    @sergeipetrov5572 2 роки тому +1

    Thanks a lot! I liked your solution of the problem!

  • @guidomartinez5099
    @guidomartinez5099 2 роки тому +3

    Nice problem. For the size contradiction, instead of splitting by cases you could start with stating that 2^(n-2) divides either m or m+1, so m+1 >= 2^(n-2) in any case, and get the contradiction. It saves a bit of case analysis.

  • @zygoloid
    @zygoloid 2 роки тому +2

    At 3:46, that is not equivalent. I think you mean

  • @krisbrandenberger544
    @krisbrandenberger544 2 роки тому

    I have decided to use induction to show that
    n=5.
    Base Case:
    5

  • @anon6514
    @anon6514 2 роки тому +1

    Nice problem.
    Here's my version of the no solutions where n > 4:
    (n-1) 2^n = (N-1) (N+1)
    For larger and larger n, the multiple of 4 on the right dominates.
    For largest max n, set:
    N+1 to be a power of 2 (so it can't eat-up any of the odd factors of n-1)
    and n to be even (so N+1 doesn't eat-up any of the even factors of n-1)
    This ensures N-1 is as large as it can be.
    Then compare:
    N-1 = 2(n-1)
    with
    N+1 = 2^(n-1)
    They differ by 2 when n = 4
    but differ by more than 2 (increasing) when n > 4

  • @owlsmath
    @owlsmath 2 роки тому +1

    Thanks! Great problem and explanation

  • @urekah37
    @urekah37 2 роки тому +3

    Thanks 😊 for making the video really needed something for my mind to do to help he rationalize.

  • @jamesfortune243
    @jamesfortune243 2 роки тому +7

    If you wrote a book with even just the major theorems used for proofs with proofs of those with examples along with a few general techniques such as strong induction, I'd buy it. I'd much prefer that to channel donations.

  • @sladebaker9882
    @sladebaker9882 2 роки тому

    Using odd numbers to find squares. Or box in a box in a box effect.
    Use N x 4 - 4

  • @Dalton1294
    @Dalton1294 2 роки тому

    n=-2 would work as plugging in that value you get .25 which is .5²

  • @alikaperdue
    @alikaperdue Рік тому

    @8:18 - the moment "= 1" is written backward in chalk on the back of Michael's shirt

  • @maxbow-arrow5931
    @maxbow-arrow5931 2 роки тому +1

    What if we allowed perfect squares of rational numbers? Are there any solutions for negative n?

  • @GrimAxel
    @GrimAxel 2 роки тому +1

    For a moment I was going to say it was satisfied for n=-2, as it *is* technically (1/2)^2 (-3*1/4+1=1/4), but then I realized they meant perfect squares in the *integers* and not the *rationals*.

  • @cortexauth4094
    @cortexauth4094 2 роки тому +1

    How does one work and see such hidden inequalities, and I loved how you could conclude om fly that m = 2^(n-2) * x
    I would have messed up there and notice it much later. I am a Computer Science and Engg student and I really wanna improve with handling such stuff in my mind, I will be glad to have suggestions. I struggle with exercises in technical/mathematical texts very bad

  • @piopod9083
    @piopod9083 2 місяці тому

    Tshirt is inside out, that proves the solution is correct.

  • @abnersosa2758
    @abnersosa2758 Рік тому

    10:58 Why doesn’t that inequation hold either for n=1 or n=4 when those values are solutions ???

  • @manucitomx
    @manucitomx 2 роки тому +2

    I’m not a fan of Number Theory, but I did enjoy this problem very much.
    Thank you, professor.

  • @Bodyknock
    @Bodyknock 2 роки тому +1

    CMU shout out! 😄

  • @thomasidzikowski1520
    @thomasidzikowski1520 2 роки тому +1

    m+1?

  • @chrisglosser7318
    @chrisglosser7318 2 роки тому

    So - rational numbers can’t be perfect squares(?!)

  • @jameyatesmauriat6116
    @jameyatesmauriat6116 2 роки тому +1

    What's the preliminary to this video by Penn to explain basics about number theory

    • @luisaleman9512
      @luisaleman9512 2 роки тому

      Check his other channel "MathMajor". He has a full course on number theory there.

  • @TonyB369
    @TonyB369 2 роки тому +1

    Isn’t the output being a fraction still possible to be a perfect square ex 1/4 = (1/2)^2
    Edit: or do perfect squares have to be integers

  • @kenbrady119
    @kenbrady119 2 роки тому +2

    Couldn't you have stopped the n≥5 case at 8:53, when the product of two integers of opposite parity (clearly odd) was "equal" to an even integer?

    • @ultimatedude5686
      @ultimatedude5686 2 роки тому +12

      Nope. 2*3 is the product of two integers of opposite parity, but it is even. In fact, the product of two numbers of opposite parity will _always_ be even, because one of the numbers must have a factor of 2.

  • @RabbidSloth
    @RabbidSloth 2 роки тому

    3:44 I'm struggling to understand why that's equivelant

  • @lucachiesura5191
    @lucachiesura5191 2 роки тому

    just a direct way

  • @tonyhaddad1394
    @tonyhaddad1394 2 роки тому

    wowww cool solution

  • @Sefra8
    @Sefra8 Рік тому

    N=-1 solution

  • @andrewdsotomayor
    @andrewdsotomayor 2 роки тому

    Does the phrase perfect square imply integers only? Is 9/49 not considered a perfect square?

    • @notananimenerd1333
      @notananimenerd1333 2 роки тому +1

      Exactly what I thought , isn’t n=-2 a solution too as for n=-2 we get the simplified value as 1/4

    • @andrewdsotomayor
      @andrewdsotomayor 2 роки тому

      @@notananimenerd1333 pretty sure the original question specified “the square of an integer” because apparently “prefect square” could mean the square of an integer or the square of a rational number, depending on the context.

    • @andrewdsotomayor
      @andrewdsotomayor 2 роки тому

      Moreover, I think it would be difficult to solve cases when n

  • @mcwulf25
    @mcwulf25 2 роки тому

    Michael is king of inequalities. Other content providers would probably use other methods to solve this.

  • @pandabearguy1
    @pandabearguy1 2 роки тому

    Its a perfect Square for all n except for the ones it is not.

  • @thapakaji8579
    @thapakaji8579 2 роки тому

    Can't we have like fractional perfect squares?

    • @Alex_Deam
      @Alex_Deam 2 роки тому

      No, the definition of perfect square is that it has an integer square root

  • @noumanegaou3227
    @noumanegaou3227 2 роки тому

    Please make a probability cours in mathmajor

  • @bentationfunkiloglio
    @bentationfunkiloglio 2 роки тому

    Very cool

  • @swingardium706
    @swingardium706 2 роки тому

    3:43 I did not follow this part. Could someone clarify?

    • @carstenmeyer7786
      @carstenmeyer7786 2 роки тому

      At that point, you want to show the following implication
      *n ≤ -2: f(n) := (n-1) * 2^n ∈ (-1; 0) => f(n) ∉ ℤ*
      Notice *f(n)* is clearly negative, so you only need to show *f(n) > -1* - that's what happens at 3:45ff.
      The reason why you do it this way is the graph of *f(x)* with *x ∈ ℝ* .

  • @General12th
    @General12th 2 роки тому

    Hi Dr.!

  • @hectormiliotis12
    @hectormiliotis12 3 місяці тому

    Ηii,has anyone solve it with square entrapment?? If yes I would like to see the solution, thank you

  • @ДавидЖелев-и6х
    @ДавидЖелев-и6х 2 роки тому

    A bit off the topic, but is 9^m-1 a perfect square?

    • @kennethforeman6164
      @kennethforeman6164 2 роки тому

      Only when m is 0. If m negative, the difference is non-integer. If m>0, a square is either 0 or 1 mod 3 but 9^m-1 is -1 mod 3.

    • @guidomartinez5099
      @guidomartinez5099 2 роки тому

      Well, 9^m is always a perfect square :). So if you subtract one it usually won't be, the only exception being m=0 as Kenneth explains.

  • @cheng-youho1441
    @cheng-youho1441 2 роки тому

    3:12 Does anyone see alternating 0 and 1 on the top of the board?

    • @amireallythatgrumpy6508
      @amireallythatgrumpy6508 2 роки тому

      Not quite alternating. There are more 0s than 1s. I think that's from the Prime Constant video.

  • @georgelaing2578
    @georgelaing2578 2 роки тому

    For all real x, we have 2 to the x power is
    greater than x.

  • @giorgibliadze1151
    @giorgibliadze1151 2 роки тому

    Please! :) can we not do induction? sick of it, already!!!! :):):) love you pro!

  • @flarklooney
    @flarklooney 2 роки тому

    @8:20 Remember, don't drink (water) and use math. 🤡